Last visit was: 29 Apr 2024, 00:24 It is currently 29 Apr 2024, 00:24

Close
GMAT Club Daily Prep
Thank you for using the timer - this advanced tool can estimate your performance and suggest more practice questions. We have subscribed you to Daily Prep Questions via email.

Customized
for You

we will pick new questions that match your level based on your Timer History

Track
Your Progress

every week, we’ll send you an estimated GMAT score based on your performance

Practice
Pays

we will pick new questions that match your level based on your Timer History
Not interested in getting valuable practice questions and articles delivered to your email? No problem, unsubscribe here.
Close
Request Expert Reply
Confirm Cancel
SORT BY:
Date
Tags:
Show Tags
Hide Tags
User avatar
Intern
Intern
Joined: 23 Dec 2012
Status:Waiting for Decisions
Posts: 33
Own Kudos [?]: 194 [56]
Given Kudos: 42
Location: India
Sahil: Bansal
GMAT 1: 570 Q49 V20
GMAT 2: 690 Q49 V34
GPA: 3
WE:Information Technology (Computer Software)
Send PM
Most Helpful Reply
User avatar
Bloomberg Exam Prep Representative
Joined: 21 Aug 2013
Posts: 24
Own Kudos [?]: 160 [5]
Given Kudos: 0
Send PM
General Discussion
User avatar
Intern
Intern
Joined: 23 Dec 2012
Status:Waiting for Decisions
Posts: 33
Own Kudos [?]: 194 [2]
Given Kudos: 42
Location: India
Sahil: Bansal
GMAT 1: 570 Q49 V20
GMAT 2: 690 Q49 V34
GPA: 3
WE:Information Technology (Computer Software)
Send PM
User avatar
Manager
Manager
Joined: 04 Oct 2013
Posts: 64
Own Kudos [?]: 292 [1]
Given Kudos: 45
Location: Brazil
GMAT 1: 660 Q45 V35
GMAT 2: 710 Q49 V38
Send PM
Re: New, strong varieties of antibiotics show the potential to [#permalink]
1
Kudos
This question is basically trying to measure your capacity to read for meaning in parallel structures. However, I suspect that the correct answer choice (E) does poorly its intent, because GMAT is very strict in terms of "which" usage. "Which" should only refer to previous term.

Read for meaning: The sentence wants to compare new varieties of antibiotics with earlier varieties. Pay attention to where the term "earlier high-strengh varieties" is located in the sentence. Any choice that refers only to one aspect of "earlier high-strengh varieties", you can eliminate it. Go ahead and cross these answers out:

New, strong varieties of antibiotics show the potential to kill a harmful bacterium without the unintended effects of killing benign bacteria and development of resistant strains of bacteria by earlier high-strength varieties.

a. unintended effects of killing benign bacteria and the development of resistant strains of bacteria by earlier high-strength varieties - linked only to the 2nd aspect
b. unintended effects by earlier high-strength varieties of development of resistant strains and killing benign bacteria - linked only to the 1st aspect
c. unintended effects for the development of resistant strains of bacteria and killing benign bacteria of earlier high-strength varieties - linked only to the 2nd aspect
d. development of resistant strains of bacteria and killing of benign bacteria that were required by earlier high-strength varieties - linked only to the 2nd aspect, no comma, no distribution
e. killing of benign bacteria and development of resistant strains of bacteria, which were unintended effects of earlier high-strength varieties - close enough to refer to both terms, but notice that "which" actually refers to a term used in the beginning of the phrase "new, strong varieties of antibiotics"



bsahil wrote:
Help needed for this question...i marked B as the answer...
Please provide explanations for options..
User avatar
VP
VP
Joined: 06 Sep 2013
Posts: 1345
Own Kudos [?]: 2393 [1]
Given Kudos: 355
Concentration: Finance
Send PM
Re: New, strong varieties of antibiotics show the potential to [#permalink]
1
Kudos
Unfortunately I picked option A here. I didn't quite like the which in answer choice E, but not too convinced either with the second part of the sentence for answer choice A

Please advice
Thanks!
Cheers
J :)
Director
Director
Joined: 22 Mar 2013
Status:Everyone is a leader. Just stop listening to others.
Posts: 611
Own Kudos [?]: 4598 [2]
Given Kudos: 235
Location: India
GPA: 3.51
WE:Information Technology (Computer Software)
Send PM
Re: New, strong varieties of antibiotics show the potential to [#permalink]
2
Kudos
1. New, strong varieties of antibiotics show the potential to kill a harmful bacterium without the unintended effects of killing benign bacteria and development of resistant strains of bacteria by earlier high-strength varieties.

5. New, strong varieties of antibiotics show the potential to kill a harmful bacterium without the killing of benign bacteria and development of resistant strains of bacteria, which were unintended effects of earlier high-strength varieties

There is a huge meaning shift in 1 AND 5.

1. without the unintended effects of killing benign bacteria
5. without the killing of benign bacteria

1 talks about effects of killing whereas 5 talks about killing of benign bacteria.

I would say its a poor quality question.
avatar
Manager
Manager
Joined: 02 Mar 2012
Posts: 201
Own Kudos [?]: 292 [0]
Given Kudos: 4
Schools: Schulich '16
Send PM
Re: New, strong varieties of antibiotics show the potential to [#permalink]
stuck between A and E.

considering whcih usage ..i marked A which is the closest.

B was temtpting but A clearly had an edge over B in my case.

Pls experts explain ....would E option be not there.....was A the answer then??


-h37
User avatar
Intern
Intern
Joined: 05 Jul 2016
Posts: 26
Own Kudos [?]: 18 [1]
Given Kudos: 129
Location: China
Concentration: Finance, Nonprofit
GMAT 1: 680 Q49 V33
GMAT 2: 690 Q51 V31
GMAT 3: 710 Q50 V36
GPA: 3.4
Send PM
Re: New, strong varieties of antibiotics show the potential to [#permalink]
1
Kudos
My approach is to eliminate those ambiguous options:

New, strong varieties of antibiotics show the potential to kill a harmful bacterium without the unintended effects of killing benign bacteria and development of resistant strains of bacteria by earlier high-strength varieties.

A. unintended effects of killing benign bacteria and the development of resistant strains of bacteria by earlier high-strength varieties
without the unintended effects of... and without the development of ...
or
without the unintended effect of ... and the development of ...
B. unintended effects by earlier high-strength varieties of development of resistant strains and killing benign bacteria
by seems illegal
Since the order of modifiers changed, this option changed the original meaning.
C. unintended effects for the development of resistant strains of bacteria and killing benign bacteria of earlier high-strength varieties
same as 2
D. development of resistant strains of bacteria and killing of benign bacteria that were required by earlier high-strength varieties
it removed unintended effects and added were required, thus changing the orginal meaning
E. killing of benign bacteria and development of resistant strains of bacteria, which were unintended effects of earlier high-strength varieties
clear and unambiguous - killing of... and development of... are two unintended effects.

So E is the best.

Thanks,
Intern
Intern
Joined: 06 Feb 2016
Posts: 21
Own Kudos [?]: 8 [0]
Given Kudos: 1380
Send PM
Re: New, strong varieties of antibiotics show the potential to [#permalink]
Hi All,

Are they parallel killing of... and development of...

Regards-
Abhijit
CR Moderator
Joined: 14 Dec 2013
Posts: 2413
Own Kudos [?]: 15269 [1]
Given Kudos: 26
Location: Germany
Schools:
GMAT 1: 780 Q50 V47
WE:Corporate Finance (Pharmaceuticals and Biotech)
Send PM
Re: New, strong varieties of antibiotics show the potential to [#permalink]
1
Kudos
Expert Reply
AbhijitGoswami wrote:
Hi All,

Are they parallel killing of... and development of...

Regards-
Abhijit


Yes, they are. A complex gerund phrase (the killing of...) can be parallel to an action noun (development).

(A simple gerund, though, cannot be parallel to an action noun. Manhattan SC guide explains the difference between simple gerund and complex gerund elaborately.)
Intern
Intern
Joined: 06 Feb 2016
Posts: 21
Own Kudos [?]: 8 [0]
Given Kudos: 1380
Send PM
Re: New, strong varieties of antibiotics show the potential to [#permalink]
Thank you Sayan. I need to revise them again :-D

Abhijit
Intern
Intern
Joined: 21 Jun 2016
Posts: 24
Own Kudos [?]: 27 [0]
Given Kudos: 116
GPA: 4
WE:Consulting (Consulting)
Send PM
Re: New, strong varieties of antibiotics show the potential to [#permalink]
I got E as it is the only option which makes the meaning of the sentence clear.

But "which" in option E refers to noun phrases and not nouns.

I thought "which" can ONLY refer to nouns.

Can the experts clarify on this?

Regards
Bitan
VP
VP
Joined: 12 Dec 2016
Posts: 1030
Own Kudos [?]: 1779 [0]
Given Kudos: 2562
Location: United States
GMAT 1: 700 Q49 V33
GPA: 3.64
Send PM
Re: New, strong varieties of antibiotics show the potential to [#permalink]
the only problem in E is "which", but as Kaplan explains, everything makes sense now.
Intern
Intern
Joined: 14 Apr 2015
Posts: 14
Own Kudos [?]: 5 [0]
Given Kudos: 13
Send PM
Re: New, strong varieties of antibiotics show the potential to [#permalink]
mikemcgarry

Could you please let me know why option E is right even though it seems to violate the which rule? i.e: which modifies the preceding word.
Magoosh GMAT Instructor
Joined: 28 Dec 2011
Posts: 4452
Own Kudos [?]: 28582 [4]
Given Kudos: 130
Re: New, strong varieties of antibiotics show the potential to [#permalink]
1
Kudos
3
Bookmarks
Expert Reply
mohitbahl wrote:
mikemcgarry

Could you please let me know why option E is right even though it seems to violate the which rule? i.e: which modifies the preceding word.

Dear mohitbahl,

I'm happy to respond. :-)

I don't have the highest opinion of this question.

What you are calling the "which rule" is something known broadly as the Modifier Touch Rule. This is an important pattern and it has important and predictable exceptions, such as appear in this SC problem. The post on the Touch has some detail and these have more:
That vs. Which on the GMAT
GMAT Grammar: Vital Noun Modifiers

Let me know if you have any questions on anything in those three blog articles.

Mike :-)
VP
VP
Joined: 12 Dec 2016
Posts: 1030
Own Kudos [?]: 1779 [0]
Given Kudos: 2562
Location: United States
GMAT 1: 700 Q49 V33
GPA: 3.64
Send PM
Re: New, strong varieties of antibiotics show the potential to [#permalink]
B,C,D are not idiom and hence out for sure.
E has better logical meaning than A does.
The key word in E is "were"; the word is plural.
Manager
Manager
Joined: 20 Jan 2016
Posts: 147
Own Kudos [?]: 128 [0]
Given Kudos: 64
Send PM
Re: New, strong varieties of antibiotics show the potential to [#permalink]
I chose option A over E because of the 'which rule'.

I see the mistake in A though now. So if you chose A like me, here is why it is wrong:

unintended effects of killing benign bacteria and the development of resistant strains of bacteria by earlier high-strength varieties

Although the sentence is parallel, the effects are by high strength varieties. But in A, it sounds like only the development is by high strength varieties which is wrong.

This error in correct in option E. Which were (plural) clearly refers to the effects(plural). I blindly rejected option E because I saw which and it cannot modify bacteria.

Please feel free to chime in if my above explanation is inaccurate.
Intern
Intern
Joined: 30 Apr 2017
Posts: 47
Own Kudos [?]: 12 [0]
Given Kudos: 74
Send PM
Re: New, strong varieties of antibiotics show the potential to [#permalink]
mikemcgarry wrote:
mohitbahl wrote:
mikemcgarry

Could you please let me know why option E is right even though it seems to violate the which rule? i.e: which modifies the preceding word.

Dear mohitbahl,

I'm happy to respond. :-)

I don't have the highest opinion of this question.

What you are calling the "which rule" is something known broadly as the Modifier Touch Rule. This is an important pattern and it has important and predictable exceptions, such as appear in this SC problem. The post on the Touch has some detail and these have more:
That vs. Which on the GMAT
GMAT Grammar: Vital Noun Modifiers

Let me know if you have any questions on anything in those three blog articles.

Mike :-)



your articles are always amazing
Thank you for such a wonderful help
Manager
Manager
Joined: 12 Apr 2018
Posts: 170
Own Kudos [?]: 137 [0]
Given Kudos: 426
Send PM
Re: New, strong varieties of antibiotics show the potential to [#permalink]
As written, the end of this sentence seems to indicate that earlier high-strength varieties of an antibiotic developed strains of bacteria resistant to the antibiotic. You need an answer choice that makes it clear that the two unintended effects, killing of benign bacteria and developing resistant strains, were encountered with the earlier versions of high-strength antibiotics.
Scan and Group the Answer Choices:
Each answer choice rearranges elements of this sentence in a different way, making it difficult to group the answer choices at a glance. Read through the choices systematically and eliminate them as you find errors.
Eliminate Wrong Answer Choices:
(A) is the sentence as written, which as discussed above is incorrect.
(B) uses the unidiomatic "unintended effects by," and (C) uses the likewise unidiomatic "unintended effects for."
(D) has a modification error. It incorrectly states that the bacteria were "required by" the earlier antibiotics. This choice also loses the information that the effects of the earlier antibiotics were "unintentional," changing the meaning of the original sentence.
(E) is the only choice that makes it clear that the two unintended effects were the effects of the earlier antibiotics. It is the correct choice.
TAKEAWAY: When awkward wording makes the meaning of a sentence unclear, look for an answer choice that removes the ambiguity and doesn't introduce any new errors.
Current Student
Joined: 15 Jun 2020
Posts: 319
Own Kudos [?]: 81 [0]
Given Kudos: 245
Location: United States
GPA: 3.3
Send PM
Re: New, strong varieties of antibiotics show the potential to [#permalink]
Hi experts,

Within D, aside from the modification error, is the parallelism wrong? "...without the development of resistant strains of bacteria and killing of benign bacteria..."

An action cannot be parallel with a simple gerund, but does the "the" at the beginning logically carry over to the "killing?" I feel like I could convince myself both ways..

MartyTargetTestPrep GMATNinja AjiteshArun
GMAT Club Bot
Re: New, strong varieties of antibiotics show the potential to [#permalink]
 1   2   
Moderators:
GMAT Club Verbal Expert
6923 posts
GMAT Club Verbal Expert
238 posts

Powered by phpBB © phpBB Group | Emoji artwork provided by EmojiOne